Question and Answers Forum

All Questions      Topic List

Others Questions

Previous in All Question      Next in All Question      

Previous in Others      Next in Others      

Question Number 91149 by Rio Michael last updated on 28/Apr/20

A particle starts from rest and moves in a straight line on a smooth   horizontal surface. Its acceleration at time t seconds is given by                                               k(4v + 1) ms^(−2)   where k is a positve constant and v ms^(−1)  is the speed of the particle.  Given that v = ((e^2 −1)/4) when t = 1.  show that                    v = (1/4)(e^(2t) −1)

$$\mathrm{A}\:\mathrm{particle}\:\mathrm{starts}\:\mathrm{from}\:\mathrm{rest}\:\mathrm{and}\:\mathrm{moves}\:\mathrm{in}\:\mathrm{a}\:\mathrm{straight}\:\mathrm{line}\:\mathrm{on}\:\mathrm{a}\:\mathrm{smooth}\: \\ $$$$\mathrm{horizontal}\:\mathrm{surface}.\:\mathrm{Its}\:\mathrm{acceleration}\:\mathrm{at}\:\mathrm{time}\:{t}\:\mathrm{seconds}\:\mathrm{is}\:\mathrm{given}\:\mathrm{by} \\ $$$$\:\:\:\:\:\:\:\:\:\:\:\:\:\:\:\:\:\:\:\:\:\:\:\:\:\:\:\:\:\:\:\:\:\:\:\:\:\:\:\:\:\:\:\:\:{k}\left(\mathrm{4}{v}\:+\:\mathrm{1}\right)\:\mathrm{ms}^{−\mathrm{2}} \\ $$$$\mathrm{where}\:{k}\:\mathrm{is}\:\mathrm{a}\:\mathrm{positve}\:\mathrm{constant}\:\mathrm{and}\:{v}\:\mathrm{ms}^{−\mathrm{1}} \:\mathrm{is}\:\mathrm{the}\:\mathrm{speed}\:\mathrm{of}\:\mathrm{the}\:\mathrm{particle}. \\ $$$$\mathrm{Given}\:\mathrm{that}\:{v}\:=\:\frac{{e}^{\mathrm{2}} −\mathrm{1}}{\mathrm{4}}\:\mathrm{when}\:{t}\:=\:\mathrm{1}.\:\:\mathrm{show}\:\mathrm{that}\: \\ $$$$\:\:\:\:\:\:\:\:\:\:\:\:\:\:\:\:\:{v}\:=\:\frac{\mathrm{1}}{\mathrm{4}}\left({e}^{\mathrm{2}{t}} −\mathrm{1}\right) \\ $$

Commented by mr W last updated on 28/Apr/20

answer given is wrong

$${answer}\:{given}\:{is}\:{wrong} \\ $$

Answered by mr W last updated on 28/Apr/20

a=(dv/dt)=k(4v+1)  (dv/(4v+1))=kdt  ∫(dv/(4v+1))=k∫dt  (1/4)ln (4v+1)=kt+C  (1/4)ln (4×((e^2 −1)/4)+1)=k+C  (1/4)ln (e^2 )=k+C  (1/4)[ln (4v+1)−ln (e^2 )]=k(t−1)  (4v+1)e^(−2) =e^(4k(t−1))   ⇒v=(1/4)[e^(4k(t−1)+2) −1]

$${a}=\frac{{dv}}{{dt}}={k}\left(\mathrm{4}{v}+\mathrm{1}\right) \\ $$$$\frac{{dv}}{\mathrm{4}{v}+\mathrm{1}}={kdt} \\ $$$$\int\frac{{dv}}{\mathrm{4}{v}+\mathrm{1}}={k}\int{dt} \\ $$$$\frac{\mathrm{1}}{\mathrm{4}}\mathrm{ln}\:\left(\mathrm{4}{v}+\mathrm{1}\right)={kt}+{C} \\ $$$$\frac{\mathrm{1}}{\mathrm{4}}\mathrm{ln}\:\left(\mathrm{4}×\frac{{e}^{\mathrm{2}} −\mathrm{1}}{\mathrm{4}}+\mathrm{1}\right)={k}+{C} \\ $$$$\frac{\mathrm{1}}{\mathrm{4}}\mathrm{ln}\:\left({e}^{\mathrm{2}} \right)={k}+{C} \\ $$$$\frac{\mathrm{1}}{\mathrm{4}}\left[\mathrm{ln}\:\left(\mathrm{4}{v}+\mathrm{1}\right)−\mathrm{ln}\:\left({e}^{\mathrm{2}} \right)\right]={k}\left({t}−\mathrm{1}\right) \\ $$$$\left(\mathrm{4}{v}+\mathrm{1}\right){e}^{−\mathrm{2}} ={e}^{\mathrm{4}{k}\left({t}−\mathrm{1}\right)} \\ $$$$\Rightarrow{v}=\frac{\mathrm{1}}{\mathrm{4}}\left[{e}^{\mathrm{4}{k}\left({t}−\mathrm{1}\right)+\mathrm{2}} −\mathrm{1}\right] \\ $$

Commented by Rio Michael last updated on 28/Apr/20

thank you sir,but i have some doubts,  first sir: at t = 1, (1/4)ln(4v + 1) = kt + C is suppose to equal      (1/4)ln(4v + 1) = k + C  don′t understand why it equals C.  also, (1/4)ln(e^2 ) = k + C  ⇒ (1/4)ln(4v + 1) = k + C  how′d you get  (1/4)[ln(4v + 1)−ln(e^2 )] = kt???

$$\mathrm{thank}\:\mathrm{you}\:\mathrm{sir},\mathrm{but}\:\mathrm{i}\:\mathrm{have}\:\mathrm{some}\:\mathrm{doubts}, \\ $$$$\mathrm{first}\:\mathrm{sir}:\:\mathrm{at}\:{t}\:=\:\mathrm{1},\:\frac{\mathrm{1}}{\mathrm{4}}\mathrm{ln}\left(\mathrm{4}{v}\:+\:\mathrm{1}\right)\:=\:{kt}\:+\:{C}\:\mathrm{is}\:\mathrm{suppose}\:\mathrm{to}\:\mathrm{equal}\: \\ $$$$\:\:\:\frac{\mathrm{1}}{\mathrm{4}}\mathrm{ln}\left(\mathrm{4}{v}\:+\:\mathrm{1}\right)\:=\:{k}\:+\:{C}\:\:\mathrm{don}'\mathrm{t}\:\mathrm{understand}\:\mathrm{why}\:\mathrm{it}\:\mathrm{equals}\:{C}. \\ $$$$\mathrm{also},\:\frac{\mathrm{1}}{\mathrm{4}}\mathrm{ln}\left({e}^{\mathrm{2}} \right)\:=\:{k}\:+\:{C}\:\:\Rightarrow\:\frac{\mathrm{1}}{\mathrm{4}}\mathrm{ln}\left(\mathrm{4}{v}\:+\:\mathrm{1}\right)\:=\:{k}\:+\:{C} \\ $$$$\mathrm{how}'\mathrm{d}\:\mathrm{you}\:\mathrm{get}\:\:\frac{\mathrm{1}}{\mathrm{4}}\left[\mathrm{ln}\left(\mathrm{4}{v}\:+\:\mathrm{1}\right)−\mathrm{ln}\left({e}^{\mathrm{2}} \right)\right]\:=\:{kt}??? \\ $$$$ \\ $$

Commented by Rio Michael last updated on 28/Apr/20

this is my approach sir,ofcourse from your method.   at rest v= 0    a = k(4v + 1) ms^(−2)  , k >0  a = (dv/dt)  ⇒  (dv/dt) = k(4v+1)    (dv/(4v+1)) = kdt ⇒ ∫(dv/(4v+1)) = k∫dt                                   (1/4)ln (4v + 1) = kt + C  at v = 0, t=0  ⇒ (1/4)ln(4(0) + 1) = C  ⇒ C=0  now t = 1 ⇒ v = ((e^2 −1)/4)  so  (1/4)ln[4(((e^2 −1)/4))+1] = k   ⇒k = 2  (1/4)ln(4v + 1) = 2k ⇒ v = (1/4)(e^(8t) −1)

$$\mathrm{this}\:\mathrm{is}\:\mathrm{my}\:\mathrm{approach}\:\mathrm{sir},\mathrm{ofcourse}\:\mathrm{from}\:\mathrm{your}\:\mathrm{method}. \\ $$$$\:\mathrm{at}\:\mathrm{rest}\:{v}=\:\mathrm{0}\: \\ $$$$\:{a}\:=\:{k}\left(\mathrm{4}{v}\:+\:\mathrm{1}\right)\:\mathrm{ms}^{−\mathrm{2}} \:,\:{k}\:>\mathrm{0} \\ $$$${a}\:=\:\frac{{dv}}{{dt}} \\ $$$$\Rightarrow\:\:\frac{{dv}}{{dt}}\:=\:{k}\left(\mathrm{4}{v}+\mathrm{1}\right) \\ $$$$\:\:\frac{{dv}}{\mathrm{4}{v}+\mathrm{1}}\:=\:{kdt}\:\Rightarrow\:\int\frac{{dv}}{\mathrm{4}{v}+\mathrm{1}}\:=\:{k}\int{dt} \\ $$$$\:\:\:\:\:\:\:\:\:\:\:\:\:\:\:\:\:\:\:\:\:\:\:\:\:\:\:\:\:\:\:\:\:\frac{\mathrm{1}}{\mathrm{4}}\mathrm{ln}\:\left(\mathrm{4}{v}\:+\:\mathrm{1}\right)\:=\:{kt}\:+\:{C} \\ $$$$\mathrm{at}\:{v}\:=\:\mathrm{0},\:{t}=\mathrm{0} \\ $$$$\Rightarrow\:\frac{\mathrm{1}}{\mathrm{4}}\mathrm{ln}\left(\mathrm{4}\left(\mathrm{0}\right)\:+\:\mathrm{1}\right)\:=\:{C} \\ $$$$\Rightarrow\:{C}=\mathrm{0} \\ $$$$\mathrm{now}\:{t}\:=\:\mathrm{1}\:\Rightarrow\:{v}\:=\:\frac{{e}^{\mathrm{2}} −\mathrm{1}}{\mathrm{4}} \\ $$$$\mathrm{so}\:\:\frac{\mathrm{1}}{\mathrm{4}}\mathrm{ln}\left[\mathrm{4}\left(\frac{{e}^{\mathrm{2}} −\mathrm{1}}{\mathrm{4}}\right)+\mathrm{1}\right]\:=\:{k}\:\:\:\Rightarrow{k}\:=\:\mathrm{2} \\ $$$$\frac{\mathrm{1}}{\mathrm{4}}\mathrm{ln}\left(\mathrm{4}{v}\:+\:\mathrm{1}\right)\:=\:\mathrm{2}{k}\:\Rightarrow\:{v}\:=\:\frac{\mathrm{1}}{\mathrm{4}}\left({e}^{\mathrm{8}{t}} −\mathrm{1}\right) \\ $$

Terms of Service

Privacy Policy

Contact: info@tinkutara.com